Question and Answers Forum

All Questions      Topic List

Limits Questions

Previous in All Question      Next in All Question      

Previous in Limits      Next in Limits      

Question Number 34516 by rahul 19 last updated on 07/May/18

lim_(x→0)  log _(tan^2 x) (tan^2 2x) = ?

$$\underset{{x}\rightarrow\mathrm{0}} {\mathrm{lim}}\:\mathrm{log}\:_{\mathrm{tan}\:^{\mathrm{2}} {x}} \left(\mathrm{tan}\:^{\mathrm{2}} \mathrm{2}{x}\right)\:=\:? \\ $$

Commented by math khazana by abdo last updated on 08/May/18

let use the changement tanx =t we have  tan^2 (2x)=( ((2tanx)/(1−tan^2 x)))^2  =(((2t)/(1−t^2 )))^2   A(x)= log_(tan^2 x) (tan^2 2x) =log_t^2   ( (((2t)/(1−t^2 ))))^2   =((2ln(((2t)/(1−t^2 ))))/(2lnt))  so   lim_(x→0) A(x) =lim_(t→0)   ((ln(2t) −ln(1−t^2 ))/(ln(t)))  =lim_(t→0)     (((1/t)  −((−2t)/(1−t^2 )))/(1/t))  (hospital theorem)  =lim_(t→0)     1 +((2t^2 )/(1−t^2 ))  =1  .

$${let}\:{use}\:{the}\:{changement}\:{tanx}\:={t}\:{we}\:{have} \\ $$$${tan}^{\mathrm{2}} \left(\mathrm{2}{x}\right)=\left(\:\frac{\mathrm{2}{tanx}}{\mathrm{1}−{tan}^{\mathrm{2}} {x}}\right)^{\mathrm{2}} \:=\left(\frac{\mathrm{2}{t}}{\mathrm{1}−{t}^{\mathrm{2}} }\right)^{\mathrm{2}} \\ $$$${A}\left({x}\right)=\:{log}_{{tan}^{\mathrm{2}} {x}} \left({tan}^{\mathrm{2}} \mathrm{2}{x}\right)\:={log}_{{t}^{\mathrm{2}} } \:\left(\:\left(\frac{\mathrm{2}{t}}{\mathrm{1}−{t}^{\mathrm{2}} }\right)\right)^{\mathrm{2}} \\ $$$$=\frac{\mathrm{2}{ln}\left(\frac{\mathrm{2}{t}}{\mathrm{1}−{t}^{\mathrm{2}} }\right)}{\mathrm{2}{lnt}}\:\:{so}\: \\ $$$${lim}_{{x}\rightarrow\mathrm{0}} {A}\left({x}\right)\:={lim}_{{t}\rightarrow\mathrm{0}} \:\:\frac{{ln}\left(\mathrm{2}{t}\right)\:−{ln}\left(\mathrm{1}−{t}^{\mathrm{2}} \right)}{{ln}\left({t}\right)} \\ $$$$={lim}_{{t}\rightarrow\mathrm{0}} \:\:\:\:\frac{\frac{\mathrm{1}}{{t}}\:\:−\frac{−\mathrm{2}{t}}{\mathrm{1}−{t}^{\mathrm{2}} }}{\frac{\mathrm{1}}{{t}}}\:\:\left({hospital}\:{theorem}\right) \\ $$$$={lim}_{{t}\rightarrow\mathrm{0}} \:\:\:\:\mathrm{1}\:+\frac{\mathrm{2}{t}^{\mathrm{2}} }{\mathrm{1}−{t}^{\mathrm{2}} }\:\:=\mathrm{1}\:\:. \\ $$

Answered by rahul 19 last updated on 07/May/18

log_(tan^2 x)  (tan^2 2x) = log_(tan x)  (tan 2x)  ⇒ ((log (tan 2x))/(log (tan x))). ((∞/∞) form)  Now applying L′ Hopital ′s rule at    lim_(x→0) ((tan 2x)/(tan x)) ⇒lim_(x→0) ((sec 2x)/(sec x))=((cos x)/(cos 2x))=1(ans.)

$${log}_{\mathrm{tan}\:^{\mathrm{2}} {x}} \:\left(\mathrm{tan}\:^{\mathrm{2}} \mathrm{2}{x}\right)\:=\:{log}_{{tan}\:{x}} \:\left(\mathrm{tan}\:\mathrm{2}{x}\right) \\ $$$$\Rightarrow\:\frac{\mathrm{log}\:\left(\mathrm{tan}\:\mathrm{2}{x}\right)}{\mathrm{log}\:\left(\mathrm{tan}\:{x}\right)}.\:\left(\frac{\infty}{\infty}\:{form}\right) \\ $$$${Now}\:{applying}\:{L}'\:{Hopital}\:'{s}\:{rule}\:{at}\: \\ $$$$\:\underset{{x}\rightarrow\mathrm{0}} {\mathrm{lim}}\frac{{tan}\:\mathrm{2}{x}}{{tan}\:{x}}\:\Rightarrow\underset{{x}\rightarrow\mathrm{0}} {\mathrm{lim}}\frac{{sec}\:\mathrm{2}{x}}{{sec}\:{x}}=\frac{\mathrm{cos}\:{x}}{\mathrm{cos}\:\mathrm{2}{x}}=\mathrm{1}\left({ans}.\right) \\ $$

Commented by MJS last updated on 07/May/18

it leads to the correct result in this case but  you cannot just put away the logarithm because  it′s not  log ((tan 2x)/(tan x))  (here you could)

$$\mathrm{it}\:\mathrm{leads}\:\mathrm{to}\:\mathrm{the}\:\mathrm{correct}\:\mathrm{result}\:\mathrm{in}\:\mathrm{this}\:\mathrm{case}\:\mathrm{but} \\ $$$$\mathrm{you}\:\mathrm{cannot}\:\mathrm{just}\:\mathrm{put}\:\mathrm{away}\:\mathrm{the}\:\mathrm{logarithm}\:\mathrm{because} \\ $$$$\mathrm{it}'\mathrm{s}\:\mathrm{not} \\ $$$$\mathrm{log}\:\frac{\mathrm{tan}\:\mathrm{2}{x}}{\mathrm{tan}\:{x}} \\ $$$$\left(\mathrm{here}\:\mathrm{you}\:\mathrm{could}\right) \\ $$

Commented by rahul 19 last updated on 07/May/18

So you mean final ans. match′ by chance′.  Otherwise in this case method is wrong?

$${So}\:{you}\:{mean}\:{final}\:{ans}.\:{match}'\:{by}\:{chance}'. \\ $$$${Otherwise}\:{in}\:{this}\:{case}\:{method}\:{is}\:{wrong}? \\ $$

Commented by MJS last updated on 07/May/18

I think so, yes. You should be careful with  this at a test...

$$\mathrm{I}\:\mathrm{think}\:\mathrm{so},\:\mathrm{yes}.\:\mathrm{You}\:\mathrm{should}\:\mathrm{be}\:\mathrm{careful}\:\mathrm{with} \\ $$$$\mathrm{this}\:\mathrm{at}\:\mathrm{a}\:\mathrm{test}... \\ $$

Answered by MJS last updated on 07/May/18

log_(tan^2  x) tan^2  2x=((ln tan^2  2x)/(ln tan^2  x))=((ln tan 2x)/(ln tan x))  (d/dx)(ln tan 2x)=2(1/(cos^2  2x))×(1/(tan 2x))=  =(2/(cos 2x sin 2x))=(1/(cos x sin x cos 2x))  (d/dx)(ln tan x)=(1/(cos^2  x))×(1/(tan x))=(1/(cos x sin x))  ((1/(cos x sin x cos 2x))/(1/(cos x sin x)))=(1/(cos 2x))  lim_(x→0) (1/(cos 2x))=1

$$\mathrm{log}_{\mathrm{tan}^{\mathrm{2}} \:{x}} \mathrm{tan}^{\mathrm{2}} \:\mathrm{2}{x}=\frac{\mathrm{ln}\:\mathrm{tan}^{\mathrm{2}} \:\mathrm{2}{x}}{\mathrm{ln}\:\mathrm{tan}^{\mathrm{2}} \:{x}}=\frac{\mathrm{ln}\:\mathrm{tan}\:\mathrm{2}{x}}{\mathrm{ln}\:\mathrm{tan}\:{x}} \\ $$$$\frac{{d}}{{dx}}\left(\mathrm{ln}\:\mathrm{tan}\:\mathrm{2}{x}\right)=\mathrm{2}\frac{\mathrm{1}}{\mathrm{cos}^{\mathrm{2}} \:\mathrm{2}{x}}×\frac{\mathrm{1}}{\mathrm{tan}\:\mathrm{2}{x}}= \\ $$$$=\frac{\mathrm{2}}{\mathrm{cos}\:\mathrm{2}{x}\:\mathrm{sin}\:\mathrm{2}{x}}=\frac{\mathrm{1}}{\mathrm{cos}\:{x}\:\mathrm{sin}\:{x}\:\mathrm{cos}\:\mathrm{2}{x}} \\ $$$$\frac{{d}}{{dx}}\left(\mathrm{ln}\:\mathrm{tan}\:{x}\right)=\frac{\mathrm{1}}{\mathrm{cos}^{\mathrm{2}} \:{x}}×\frac{\mathrm{1}}{\mathrm{tan}\:{x}}=\frac{\mathrm{1}}{\mathrm{cos}\:{x}\:\mathrm{sin}\:{x}} \\ $$$$\frac{\frac{\mathrm{1}}{\mathrm{cos}\:{x}\:\mathrm{sin}\:{x}\:\mathrm{cos}\:\mathrm{2}{x}}}{\frac{\mathrm{1}}{\mathrm{cos}\:{x}\:\mathrm{sin}\:{x}}}=\frac{\mathrm{1}}{\mathrm{cos}\:\mathrm{2}{x}} \\ $$$$\underset{{x}\rightarrow\mathrm{0}} {\mathrm{lim}}\frac{\mathrm{1}}{\mathrm{cos}\:\mathrm{2}{x}}=\mathrm{1} \\ $$

Commented by rahul 19 last updated on 07/May/18

Thank you sir !

Terms of Service

Privacy Policy

Contact: info@tinkutara.com